Solving Redshift of Two Kinds in Schwarzschild Metric

  • Thread starter WannabeNewton
  • Start date
  • Tags
    Redshift
In summary, the conversation discusses a problem involving a spaceship on a circular orbit emitting a photon at an angle outward from the tangential direction of motion, and the frequency of the photon as seen by a stationary observer at a large distance. The conversation also considers the same scenario with a radially infalling observer. The calculation results in a ratio of the received wavelength to the emitted wavelength, and the correct answer should satisfy ##\frac{\lambda_{\infty}}{\lambda} \propto (1 - 2M/r)^{-1}##. However, the calculation done in the conversation results in ##\frac{\lambda_{\infty}}{\lambda} = (1 - \sqrt{2M/r})^{-1}
  • #1
WannabeNewton
Science Advisor
5,844
551
Redshift of two "kinds"

Hello! Consider the following problem: A spaceship on a circular orbit at radius ##r## in a Schwarzschild metric emits a photon with the rest frame frequency ##\omega## at an angle ##\alpha## outward from the tangential direction of the motion, in the plane of the orbit. What is the frequency of the photon as seen by a stationary observer at a large distance?

Call the spaceship frame ##O## and imagine that at the emission event there is a stationary frame ##O'##. At that event, ##O## has axes ##e_{\hat{x}}, e_{\hat{y}}## in the plane of the orbit with ##e_{\hat{x}}## pointing along the tangential direction of the orbit at that event. The wave 4-vector at this event of the emitted photon as represented in ##O## is just ##k = \omega_0 e_{\hat{t}} + \omega_0\cos\alpha e_{\hat{x}} + \omega_0 \sin\alpha e_{\hat{y}}##. Furthermore, the speed of ##O## relative to ##O'## is given by ##v = \sqrt{\frac{M/r}{1 - 2M/r}}## (this can easily be derived by using the fact that ##(u^{\mu})_{O}(u_{\mu})_{O'} = -\frac{1}{\sqrt{1 - v^2}}##). Thus, using an inverse Lorentz boost in the ##e_{\hat{x}}## direction to go from ##O## to ##O'## at this event, we have the frequency ##\omega'## as measured in ##O'##: ##\omega' = k^{0'} = \Lambda^{0'}{}{}_{\nu}k^{\nu} = \omega\gamma(1 + v\cos\alpha)##. Therefore ##\omega_{\infty} = (1 - 2M/r)^{1/2}\omega' = \omega\gamma(1 + v\cos\alpha)(1 - 2M/r)^{1/2}## which is what my text lists as the correct answer.

Now I know this may seem like a roundabout way of solving this problem but I did it this way for a particular reason. Consider now the scenario wherein an observer with frame ##O''## falls radially inwards from rest at infinity. At some ##r## this observer emits a photon, with frequency ##\omega## in ##O''##, radially outwards which is received by a static observer at infinity. Consider again a static frame ##O'## at the emission event. At this event ##O''## has a radial axis ##e_{\hat{r}}## in the direction of motion. The photon wave 4-vector at emission as represented in ##O''## is ##k = \omega e_{\hat{t}} - \omega e_{\hat{r}}##. The speed of ##O''## relative to ##O'## is ##v = \sqrt{2M/r}## so performing an inverse Lorentz boost in the ##e_{\hat{r}}## direction to go from ##O''## to ##O'## we have ##\omega' = \gamma \omega (1 - v)## hence ##\omega_{\infty} = (1 - 2M/r)^{1/2}\omega' = \gamma \omega (1 - v)(1 - 2M/r)^{1/2} = \omega(1 - v) = \omega(1 - \sqrt{2M/r})## since ##\gamma = (1 - 2M/r)^{-1/2}##.

But this is nowhere near close to the answer, which should be ##\omega_{\infty} = \omega(1 - 2M/r)## so where did I go wrong? Why did this method work for the case of the circular orbit but not for the case of the radial free fall from rest at infinity?

By the way I should note that I did the radial case in the direct way and still didn't get the right answer. By "direct way" I mean simply evaluating ##\omega = -k_{\mu}(u^{\mu})_{O''}## in the Schwarzschild basis, using the fact that ##u_{O''} = (1 - 2M/r)^{-1}\partial_{t} - \sqrt{2M/r}\partial_{r}##. For the wave 4-vector ##k = k^t \partial_t + k^r \partial_r## we have that ##k^{r}\rightarrow 1## as ##r \rightarrow \infty## which fixes the conserved energy ##e = k^r = 1## hence ##k^t = (1 - 2M/r)^{-1}##. Therefore ##\omega = (1 - 2M/r)^{-1}(1 + \sqrt{2M/r}) = (1 - \sqrt{2M/r})^{-1}## hence ##\omega_{\infty} = 1 = \omega(1 - \sqrt{2M/r} )## which is the same result as above so I still don't get where I'm going wrong.

Thanks in advance!
 
Physics news on Phys.org
  • #2
WannabeNewton said:
But this is nowhere near close to the answer, which should be ##\omega_{\infty} = \omega(1 - 2M/r)## so where did I go wrong?
Is this the textbook answer? I cannot fault your arguments or calculations so maybe you are "not wrong".
 
  • #3
WannabeNewton said:
we have ##\omega' = \gamma \omega (1 - v)## hence ##\omega_{\infty} = (1 - 2M/r)^{1/2}\omega' = \gamma \omega (1 - v)(1 - 2M/r)^{1/2} = \omega(1 - v) = \omega(1 - \sqrt{2M/r})## since ##\gamma = (1 - 2M/r)^{-1/2}##.

But this is nowhere near close to the answer, which should be ##\omega_{\infty} = \omega(1 - 2M/r)##

According to whom? I think that, except for a possible sign error, your answer is correct.

WannabeNewton said:
The photon wave 4-vector at emission as represented in ##O''## is ##k = \omega e_{\hat{t}} - \omega e_{\hat{r}}##.

Since the photon is moving radially outwards, shouldn't this be ##k = \omega e_{\hat{t}} + \omega e_{\hat{r}}##?

Using simplistic methods, I got the same result as you, except for a sign difference. See

https://www.physicsforums.com/showthread.php?p=4373165#post4373165
 
  • #4
George Jones said:
According to whom? I think that, except for a possible sign error, your answer is correct.

yuiop said:
Is this the textbook answer? I cannot fault your arguments or calculations so maybe you are "not wrong".

Thanks for the replies! If I read the passage correctly then yes this is what the text says *should* be the answer. More specifically, this is exercise 8.4(a) on page 356 of Padmanabhan's GR text (which I know George has). It says that the ratio of the received wavelength to the emitted wavelength should satisfy ##\frac{\lambda_{\infty}}{\lambda} \propto (1 - 2M/r)^{-1}## whereas I have ##\frac{\lambda_{\infty}}{\lambda} = (1 - \sqrt{2M/r})^{-1} = (1 - 2M/r)^{-1}(1 + \sqrt{2M/r}) ##.

George Jones said:
Since the photon is moving radially outwards, shouldn't this be ##k = \omega e_{\hat{t}} + \omega e_{\hat{r}}##?

Using simplistic methods, I got the same result as you, except for a sign difference. See

https://www.physicsforums.com/showthread.php?p=4373165#post4373165
I had ##e_{\hat{r}}## pointing in the direction of motion of the radially infalling observer so wouldn't the outgoing light ray be pointing in the ##-e_{\hat{r}}## direction? I got the same sign when I did it the second way (last paragraph of post #1) wherein I didn't use a local Lorentz frame so I don't know if I'm making some kind of algebraic mistake.

Also if ##\frac{\lambda_{\infty}}{\lambda} = 1 + \sqrt{2M/r} ## as in the link then wouldn't we fail to get ##\frac{\lambda_{\infty}}{\lambda} \rightarrow \infty ## as ##r \rightarrow 2M##? Whereas if ##\frac{\lambda_{\infty}}{\lambda} = (1 - \sqrt{2M/r})^{-1}## then ##\frac{\lambda_{\infty}}{\lambda} \rightarrow \infty## as ##r \rightarrow 2M##. Shouldn't we have infinite redshift of the light ray that is sent out to the distant static observer right when the radially infalling observer reaches the event horizon?

Thanks again for the replies!
 
  • #5
WannabeNewton said:
Thanks for the replies! If I read the passage correctly then yes this is what the text says *should* be the answer. More specifically, this is exercise 8.4(a) on page 356 of Padmanabhan's GR text (which I know George has). It says that the ratio of the received wavelength to the emitted wavelength should satisfy ##\frac{\lambda_{\infty}}{\lambda} \propto (1 - 2M/r)^{-1}##

Yes, this is the way I read it.

WannabeNewton said:
I had ##e_{\hat{r}}## pointing in the direction of motion of the radially infalling observer so wouldn't the outgoing light ray be pointing in the ##-e_{\hat{r}}## direction?

Okay. I thought the notation ##e_{\hat{r}}## indicated a unit vector in the ##\partial_r## direction.

WannabeNewton said:
I got the same sign when I did it the second way (last paragraph of post #1) wherein I didn't use a local Lorentz frame so I don't know if I'm making some kind of algebraic mistake.

Also if ##\frac{\lambda_{\infty}}{\lambda} = 1 + \sqrt{2M/r} ## as in the link then wouldn't we fail to get ##\frac{\lambda_{\infty}}{\lambda} \rightarrow \infty ## as ##r \rightarrow 2M##? Whereas if ##\frac{\lambda_{\infty}}{\lambda} = (1 - \sqrt{2M/r})^{-1}## then ##\frac{\lambda_{\infty}}{\lambda} \rightarrow \infty## as ##r \rightarrow 2M##. Shouldn't we have infinite redshift of the light ray that is sent out to the distant static observer right when the radially infalling observer reaches the event horizon?

I think so. I now realize that we calculated different things. My calculation shows that an observer falling freely from rest at infinity doesn't experience infinite blueshift at the event horizon.

I probably won't have another chance to look at this until tomorrow morning west coast time (parent-teacher meeting, daughter's soccer, etc.)
 
  • #6
Some last comments before I leave work.

First, I found a book that gives both our results,

Also, I noticed something interesting when our two results are combined.

Suppose an observer at infinity emits wavelength ##\lambda_1## light in a radially inwards direction. This light is received at ##r## with wavelength ##\lambda_2## by an observer, Fred, who falls freely from infinity. Fred immediately emits light of the same wavelength (##\lambda_2##) in a radially outwards direction. This light is observed with wavelength ##\lambda_3## at infinity.

What is the relationship between ##\lambda_1##, ##\lambda_3##, and ##r##?
 
  • #7
George Jones said:
Yes, this is the way I read it.
Cool, so now that just leaves the question of how Padmanabhan gets that result, in light of the above.

George Jones said:
Okay. I thought the notation ##e_{\hat{r}}## indicated a unit vector in the ##\partial_r## direction.
Ah ok, sorry about the confusion. I should have used some other notation.

George Jones said:
I probably won't have another chance to look at this until tomorrow morning west coast time (parent-teacher meeting, daughter's soccer, etc.)

Have fun!

George Jones said:
Some last comments before I leave work.

First, I found a book that gives both our results,
Yey! And what book is that, if you don't mind me asking?

George Jones said:
Suppose an observer at infinity emits wavelength ##\lambda_1## light in a radially inwards direction. This light is received at ##r## with wavelength ##\lambda_2## by an observer, Fred, who falls freely from infinity. Fred immediately emits light of the same wavelength (##\lambda_2##) in a radially outwards direction. This light is observed with wavelength ##\lambda_3## at infinity.

What is the relationship between ##\lambda_1##, ##\lambda_3##, and ##r##?

Well using what you had, ##\frac{\lambda_2}{\lambda_1} = 1 + \sqrt{2M/r}##, and the above result that ##\frac{\lambda_3}{\lambda_2} = (1 - \sqrt{2M/r})^{-1}## we would get ##\frac{\lambda_3}{\lambda_1} = \frac{\lambda_3}{\lambda_2}\frac{\lambda_2}{\lambda_1} = \frac{1 + \sqrt{2M/r}}{1 - \sqrt{2M/r}}## hence ##\lambda_3 > \lambda_1##; does this agree with what you had?
 
  • #8
WannabeNewton said:
More specifically, this is exercise 8.4(a) on page 356 of Padmanabhan's GR text (which I know George has). It says that the ratio of the received wavelength to the emitted wavelength should satisfy ##\frac{\lambda_{\infty}}{\lambda} \propto (1 - 2M/r)^{-1}## whereas I have ##\frac{\lambda_{\infty}}{\lambda} = (1 - \sqrt{2M/r})^{-1} = (1 - 2M/r)^{-1}(1 + \sqrt{2M/r}) ##.
Suspiciously, Padmanabhan's answer uses the 'proportional to' symbol rather than the equality symbol, implying some factor is missing and it would seem that factor is your ##(1 + \sqrt{2M/r}) ##. I have checked out both George's and your solutions and they appear to be correct (including signs) for the slightly different contexts they are used in.
 
  • #9
yuiop said:
Suspiciously, Padmanabhan's answer uses the 'proportional to' symbol rather than the equality symbol, implying some factor is missing and it would seem that factor is your ##(1 + \sqrt{2M/r}) ##

That's what I wanted to think but usually the proportionality symbol is written for constants whereas here the ##(1 + \sqrt{2M/r}) ## factor is variable with the point of emission ##r##.
 
  • #10
WannabeNewton said:
That's what I wanted to think but usually the proportionality symbol is written for constants whereas here the ##(1 + \sqrt{2M/r}) ## factor is variable with the point of emission ##r##.
I agree that the missing factor is *usually* a constant. I was being generous to the author. :-p
 
  • #11
Padmanabhan should have started his question with "(a) In the asymptotic limit as ##r \rightarrow 2M_+##, compute ..."

In this asymptotic limit, your result and Padmanabhan's result agree. For example, write ##2M/r = 1 -\epsilon##.

I was motivated to try this by Straumann's statement: "we obtain the asymptotic expression

$$1 + z \simeq \frac{4ME}{r - 2M}.$$
 
  • #12
Ah ok I see, thank you so much for the help George! So does that mean that the exponential relationship ##\lambda_{\text{rec}}/\lambda_{\text{em}} \propto e^{t_{\text{rec}}/4M}## is really only valid when the infalling observer is very close to the event horizon (part b of the exercise)?
 
  • #13
WannabeNewton said:
So does that mean that the exponential relationship ##\lambda_{\text{rec}}/\lambda_{\text{em}} \propto e^{t_{\text{rec}}/4M}## is really only valid when the infalling observer is very close to the event horizon (part b of the exercise)?

Yes.
 
  • Like
Likes 1 person

Related to Solving Redshift of Two Kinds in Schwarzschild Metric

1. What is the Schwarzschild metric and why is it important in solving redshift?

The Schwarzschild metric is a mathematical solution to Einstein's field equations in general relativity, which describes the gravitational field outside a spherically symmetric, non-rotating mass. It is important in solving redshift because it allows us to calculate the observed shift in the wavelength of light emitted from an object due to the effects of gravity, such as the curvature of space-time around a massive object like a black hole.

2. How is redshift of two kinds measured using the Schwarzschild metric?

The redshift of two kinds can be measured by comparing the observed wavelength of light from a source at a distance with its expected wavelength in the absence of gravity. The Schwarzschild metric provides a formula for calculating the expected wavelength, which can then be compared to the observed wavelength to determine the amount of redshift.

3. Can the Schwarzschild metric be used to solve redshift in all situations?

No, the Schwarzschild metric is only applicable in situations where the mass causing the gravitational field is spherically symmetric and non-rotating. In other cases, more complex metrics and equations must be used to accurately calculate redshift.

4. How does the Schwarzschild metric account for the two types of redshift?

The Schwarzschild metric takes into account two types of redshift: the gravitational redshift, which is caused by the curvature of space-time around a massive object, and the cosmological redshift, which is caused by the expansion of the universe. The metric includes terms that account for both of these effects in calculating the observed wavelength of light.

5. Are there any limitations or uncertainties in using the Schwarzschild metric to solve redshift?

Yes, there are limitations and uncertainties in using the Schwarzschild metric to solve redshift. One limitation is that it assumes a static gravitational field, which is not always the case. Additionally, there may be uncertainties in the mass and distance measurements of the objects being observed, which can affect the accuracy of the calculated redshift. Furthermore, the metric does not account for other factors that may contribute to redshift, such as the Doppler effect.

Similar threads

  • Special and General Relativity
Replies
8
Views
1K
  • Special and General Relativity
Replies
1
Views
2K
  • Special and General Relativity
Replies
28
Views
2K
  • Special and General Relativity
2
Replies
42
Views
3K
  • Special and General Relativity
Replies
11
Views
519
  • Special and General Relativity
Replies
4
Views
1K
  • Special and General Relativity
2
Replies
43
Views
2K
  • Special and General Relativity
Replies
20
Views
5K
  • Introductory Physics Homework Help
Replies
2
Views
897
  • Special and General Relativity
Replies
1
Views
951
Back
Top